Why are Bell's inequalities violated?

In summary, the conversation discusses the reasons for the violation of Bell's inequalities, which are based on the measurement of non-commuting quantum observables. It is noted that the experiments themselves show that the underlying "hidden variables" may interact with the measuring device in an unknown physical way, leading to a change in values. This challenges the assumption that the measurement of one observable does not affect the value of another, which is the basis of Bell's inequality. The conversation also mentions a paper by A. Peres, which further explores this concept and concludes that there is no way to account for the predictions of quantum mechanics using hypothetical unobserved experiments.
  • #1
JK423
Gold Member
394
7
Hello guys,

I am trying hard to understand the reason of the violation, and i hope you give me some help.

Here is my understanding so far:
Bell's inequalities are based on the measurement of non-commuting quantum observables, e.g. the measurement of the spin in x and z direction. This, to start with, raises red flags! That's because the proof of Bell's inequalities does not include a dependence on what observable we measure, instead it's assumed that both sx and sz have definite values and are not affected by the measurement.
But the experiments themselves, already tell us that this is not true! If you measure sx then the value of sz is altered! What this fact could mean is that the underlying "hidden variables" interact with the measuring device in an unknown physical way.
For example, throw an electron in a Stern-Gerlach aparratus (measuring sz )and assume that this electron has well defined spins in all directions x,y,z before the interaction, described by an underlying local & realistic hidden variable theory. Ok, now the electron is seen to go upwards, i.e. it has sz =+1. However, this interaction with the magnetic field may have altered sx and sy in an uncontrollable way! So, even if we had previously measured sx, its new value after the measurement of sz is different due to unknown underlying local, realistic physics!
That way, it seems quite obvious that Bell's inequality may be violated without assuming non-locality or absense of reality, since the derivation of Bell's inequality is based on the assumption that the measurement of sz does not change the value of sx.

What is your opinion on this? In the literature, has it been studied? Are there any physical arguments against it?

Thank you a lot!

Giannis
 
Last edited:
Physics news on Phys.org
  • #2
JK423 said:
Here is my understanding so far:
Bell's inequalities are based on the measurement of non-commuting quantum observables, e.g. the measurement of the spin in x and z direction. This, to start with, raises red flags! That's because the proof of Bell's inequalities does not include a dependence on what observable we measure, instead it's assumed that both sx and sz have definite values and are not affected by the measurement.
But the experiments themselves, already tell us that this is not true! If you measure sx then the value of sz is altered! What this fact could mean is that the underlying "hidden variables" interact with the measuring device in an unknown physical way.

For example, throw an electron in a Stern-Gerlach aparratus (measuring sz )and assume that this electron has well defined spins in all directions x,y,z before the interaction, described by an underlying local & realistic hidden variable theory. Ok, now the electron is seen to go upwards, i.e. it has sz =+1. However, this interaction with the magnetic field may have altered sx and sy in an uncontrollable way! So, even if we had previously measured sx, its new value after the measurement of sz is different due to unknown underlying local, realistic physics!

That way, it seems quite obvious that Bell's inequality may be violated without assuming non-locality or absense of reality, since the derivation of Bell's inequality is based on the assumption that the measurement of sz does not change the value of sx.

This is the entire point of the Bell argument, and is well considered. Suppose what you say is correct. If so, then there are values for what the outcome would be IF you had measured at other angle settings. It turns out that you cannot come up with a set of those values in which the results here are independent of the setting there!

The easiest example to see this is with entangled photon pairs in which their polarization is always alike (ie from Type I down conversion crystals). Use the 3 angles 0, 120, 240 degrees. These are selected because the relative difference between any pair of these angle setting is either 0 degrees (same setting for Alice and Bob) or 120 degrees (any different setting for Alice and Bob). If you have a local realistic theory, the result for Bob cannot depend on the setting for distant Alice, right?

According to QM, the coincidence rate for Alice and Bob, when they compare their results, will be .25 (25%) which is cos^2(120 degrees). Of course this is when Alice and Bob independently choose different angle settings. When they happen to choose the same setting, they must ALWAYS get the same result.

Are you with me so far?
 
  • #3
DrChinese said:
If so, then there are values for what the outcome would be IF you had measured at other angle settings. It turns out that you cannot come up with a set of those values in which the results here are independent of the setting there!

Correct.
A little while after my original post, i found a paper of A. Peres, "Unperformed experiments have no results" (Am. J. Phys. 46(7), July 1978), where he describes exactly this point.
So you are right, there are two kinds of results entering a Bell inequality: the actual results from experiments that were performed, and the hypothetical results coming from experiments that were never performed. And there is no possible way to fill the latter in order to account for the quantum mechanical predictions.

That's very nice :)
 
  • #4
JK423 said:
Correct.
A little while after my original post, i found a paper of A. Peres, "Unperformed experiments have no results" (Am. J. Phys. 46(7), July 1978), where he describes exactly this point.
So you are right, there are two kinds of results entering a Bell inequality: the actual results from experiments that were performed, and the hypothetical results coming from experiments that were never performed. And there is no possible way to fill the latter in order to account for the quantum mechanical predictions.

That's very nice :)

Yes, and keep in mind that Quantum Mechanics does NOT make the statement that the hypothetical unobserved (counterfactual) experiments have results. This is exclusively in the realm of the local realistic theory. Really, a Bell test is simply confirmation of the cos^2(theta) prediction of QM and nothing more. The Bell Inequality is simply a mechanism for showing that there exist situations in which a local realistic theory would make nonsensical predictions if QM IS correct.
 
  • #5
What do you mean "if QM IS correct"?
The experiments violate the local realistic predictions irrespectively of QM's validity, right?
 
Last edited:
  • #6
JK423 said:
What do you mean "if QM IS correct"?
The experiments violate the local realistic predictions irrespectively of QM's validity, right?

Yes, true. The inequalities are usually oriented (and experiments designed) so that local realistic (LR) theories are on one side and QM on the other. But that is not an absolute requirement. Technically, both LR and QM could be wrong.
 
  • #7
JK423 said:
Hello guys,

I am trying hard to understand the reason of the violation, and i hope you give me some help.

Here is my understanding so far:
Bell's inequalities are based on the measurement of non-commuting quantum observables, e.g. the measurement of the spin in x and z direction. This, to start with, raises red flags! That's because the proof of Bell's inequalities does not include a dependence on what observable we measure, instead it's assumed that both sx and sz have definite values and are not affected by the measurement.
But the experiments themselves, already tell us that this is not true! If you measure sx then the value of sz is altered! What this fact could mean is that the underlying "hidden variables" interact with the measuring device in an unknown physical way.
For example, throw an electron in a Stern-Gerlach aparratus (measuring sz )and assume that this electron has well defined spins in all directions x,y,z before the interaction, described by an underlying local & realistic hidden variable theory. Ok, now the electron is seen to go upwards, i.e. it has sz =+1. However, this interaction with the magnetic field may have altered sx and sy in an uncontrollable way! So, even if we had previously measured sx, its new value after the measurement of sz is different due to unknown underlying local, realistic physics!
That way, it seems quite obvious that Bell's inequality may be violated without assuming non-locality or absense of reality, since the derivation of Bell's inequality is based on the assumption that the measurement of sz does not change the value of sx.

What is your opinion on this? In the literature, has it been studied? Are there any physical arguments against it?

Thank you a lot!

Giannis
You're on the wrong track. As far as I'm aware your approach has been explored and doesn't lead to understanding what it is that makes Bell's formulation incompatible with qm and experiment.
There's no argument that some property (or properties) of the incident disturbances in relation to analyzer orientation determines individual detection, or that this property exists before, and is changed by, interaction with the analyzers.

Bell's theorem has to do with how an lhv model of quantum entanglement might be written.
There's still no universally accepted answer as to why it can't take the form that Bell proposed, or why Bell inequalities are violated.

Maybe it's that nature really is nonlocal and this nonlocality is manifested uniquely in Bell tests. This seems unlikely and a bit too convenient for some.
Maybe it has to do with the way the functions for individual detection are combined.
Maybe Bell's locality condition encodes a restriction that has nothing to do with locality.

These approaches, and more, are being explored. No definitive answer yet. Your thread question is an open question in physics.
 
  • #8
We don't know if Bell's inequality is going to be violated in a loophole free test.

Taking the conclusion from one experiment that closes x loophole, and coupling that with the conclusion from another experiment that closes y loophole doesn't mean it is going to hold if one experiment closes x and y loopholes.

Though it seems unlikely, the physics community seems to be jumping up and down over non-locality being real when no such test confirms it.
 
  • #9
StevieTNZ said:
We don't know if Bell's inequality is going to be violated in a loophole free test.

Taking the conclusion from one experiment that closes x loophole, and coupling that with the conclusion from another experiment that closes y loophole doesn't mean it is going to hold if one experiment closes x and y loopholes.

Though it seems unlikely, the physics community seems to be jumping up and down over non-locality being real when no such test confirms it.

You don't require the same level of loophole-free scientific proof for anything else, why for Bell? (That's a rhetorical question. :smile: )

I would certainly be interested in hearing about a local realistic theory that fails when locality is maintained, AND fails when the full universe is tested, but succeeds when both locality is maintained AND the full universe is tested at the same time. Exactly how do you think that would work?

Oh, and how is it that QM is so wrong for every type of entanglement ever tested so far? That is, considering there is no such thing as entanglement in a local realistic world?
 
  • #10
StevieTNZ said:
We don't know if Bell's inequality is going to be violated in a loophole free test.

We don't, but that's true of every experimental test of every proposition.

At this point, the odds of finding a loophole (in Bell's argument and the experiments that support the violation of the equality) that would allow us to bring back a local realistic theory of the sort that EPR hoped for are pretty slim.
 
  • #11
nanosiborg said:
Maybe it's that nature really is nonlocal and this nonlocality is manifested uniquely in Bell tests. This seems unlikely and a bit too convenient for some.
Why do you think it's unlikely and too convenient?
 
  • #12
DrChinese said:
Oh, and how is it that QM is so wrong for every type of entanglement ever tested so far? That is, considering there is no such thing as entanglement in a local realistic world?

We don't know if QM is correct, if the loopholes are all closed in one experiment. We only know QM is correct is we allow certain loopholes.
 
  • #13
DrChinese said:
You don't require the same level of loophole-free scientific proof for anything else, why for Bell? (That's a rhetorical question. :smile: )

How do you know I don't?
 
  • #14
StevieTNZ said:
We don't know if QM is correct, if the loopholes are all closed in one experiment. We only know QM is correct is we allow certain loopholes.

We DO know for ALL tests performed to date. Same as for general relativity, our theories of galaxy formation and astrophysics in general, chemistry, atomic structure, evolution, standard model, etc etc etc.

As I said, there is no scientific reason to mention that loophole-free tests of ALL theories have never been performed. No one is claiming we know everything. All we are claiming is that if QM is correct, there are no local hidden variable theories. And every test supports QM, which is all you can say for any theory.

In normal everyday speak: a theory with sound experimental support is called "CORRECT".
 
  • #15
Nugatory said:
At this point, the odds of finding a loophole (in Bell's argument and the experiments that support the violation of the equality) that would allow us to bring back a local realistic theory of the sort that EPR hoped for are pretty slim.

True. About as likely as discovering that relativity doesn't exist. Then we would wake up in a Newtonian local realistic universe*.

*I am guessing one that is also about 4000 years old. :biggrin:
 
  • #16
bohm2 said:
Why do you think it's unlikely and too convenient?
Unlikely because it's an assumption without evidence. Convenient because, using Bell's formulation, assuming nonlocality allows that the results at one end depend on the analyzer settings at the other end. Too convenient for my taste. Maybe not yours and others. But at this point it is just a matter of taste.

I think that something other than nonlocality will eventually answer the thread question.
 
  • #17
StevieTNZ said:
How do you know I don't?

Because we don't see you over in the relativity forum worrying about loopholes in the M-M experiments? :smile:
 
  • Like
Likes Derek Potter
  • #18
DrChinese said:
You don't require the same level of [..] scientific proof for anything else, why for Bell? (That's a rhetorical question. :smile: ) [..]
Some rhetorical questions deserve an answer. Extreme claims require extreme evidence. Thus I require the same level of scientific proof as for perpetuum mobilae. :tongue:
 
  • #19
nanosiborg said:
I think that something other than nonlocality will eventually answer the thread question.
There seems to be only 3 options based on assumptions made by Bell:

1. Non-locality
2. Anti-realism
3. Superdeterminism (no freedom of choice)
 
Last edited:
  • #20
harrylin said:
Some rhetorical questions deserve an answer. Extreme claims require extreme evidence. Thus I require the same level of scientific proof as for perpetuum mobilae. :tongue:

Well, first of all, the observed results match theory. That isn't true of perpetual motion machines.

Second, I guess you answered the question about why you don't demand the same proof for relativity. The answer is that what is "extreme" is subjective (to you). You consider relativity "reasonable" in light of experimental proof but falsification of local realism "unreasonable" in light of experimental proof. Ergo you essentially conclude that which you sought to prove.

The reason I called it a rhetorical question is because of this point. If you are a local realist in 2013, you aren't going to let evidence affect your viewpoint. So no point in trying to answer the question.
 
  • #21
DrChinese said:
Well, first of all, the observed results match theory. That isn't true of perpetual motion machines.

Second, I guess you answered the question about why you don't demand the same proof for relativity. The answer is that what is "extreme" is subjective (to you). You consider relativity "reasonable" in light of experimental proof but falsification of local realism "unreasonable" in light of experimental proof. Ergo you essentially conclude that which you sought to prove.

The reason I called it a rhetorical question is because of this point. If you are a local realist in 2013, you aren't going to let evidence affect your viewpoint. So no point in trying to answer the question.
For sure perpetual motion machines match the theory of such experimenters, or so they claim (I did look into a few of them, so I know what I'm talking about). And not surprisingly, those experimenters reply in quite the same manner, claiming that no evidence is ever good enough for the scientific community - however, that's not true; only the requirements are extremely strict. But thanks for clarifying why your question was rhetorical. :smile:
 
  • #22
harrylin said:
For sure perpetual motion machines match the theory of such experimenters, or so they claim (I did look into a few of them, so I know what I'm talking about).

Ah, that is still not the same at all. Bell involves no modification to preexisting, accepted theory that forms our common ground. (I assume you don't disagree with QM pre-Bell.) So you can't really object that it hypothesizes new effects. There aren't any. The only new thing is realizing that QM did predict entanglement, and thus you can have a field day with experiments in that vein that are derived from orthodox QM. Of course, orthodox QM itself is falsifiable on a variety of fronts.

On the other hand, PMM advocates ARE proposing modifications to pre-existing, accepted theory (so there is not common ground). Those should be falsifiable if they are to be useful (otherwise they would be "ad hoc").

In my book, you pick and choose what evidence you accept, in order to be consistent with your pre-ordained conclusion.
 
  • #23
Another question came up..
The CHSH quantity,
[itex]S_j = A_j\left( {{a_1}} \right)B_j\left( {{b_1}} \right) + A_j\left( {{a_1}} \right)B_j\left( {{b_2}} \right) + A_j\left( {{a_2}} \right)B_j\left( {{b_1}} \right) - A_j\left( {{a_2}} \right)B_j\left( {{b_2}} \right)[/itex],
where [itex]A_j\left( {{a_i}} \right) = \pm 1[/itex] and [itex]B_j\left( {{b_i}} \right) = \pm 1[/itex], and j denoting a particular photon pair,
is always [itex]{S_j} = \pm 2[/itex], for any measurement result A and B.
When we take the mean value over all photon pairs, [itex]\,\left\langle S \right\rangle = \frac{1}{N}\sum\limits_{i = 1}^N {{S_j}} [/itex] we find it to be bounded, i.e.
[itex] - 2 \le \,\left\langle S \right\rangle \le 2[/itex].
This quantity is bounded whatever the values of A and B for the photon pairs.

Say that the choice of the angles [itex]a_i[/itex],[itex]b_i[/itex] is not random but they are correlated to each other. I don't see how this inequality could be violated by a local and realistic model.. Can you help? I am trying to understand how this *loophole* works..
 
  • #24
JK423 said:
Another question came up..
The CHSH quantity,
[itex]S_j = A_j\left( {{a_1}} \right)B_j\left( {{b_1}} \right) + A_j\left( {{a_1}} \right)B_j\left( {{b_2}} \right) + A_j\left( {{a_2}} \right)B_j\left( {{b_1}} \right) - A_j\left( {{a_2}} \right)B_j\left( {{b_2}} \right)[/itex],
where [itex]A_j\left( {{a_i}} \right) = \pm 1[/itex] and [itex]B_j\left( {{b_i}} \right) = \pm 1[/itex], and j denoting a particular photon pair,
is always [itex]{S_j} = \pm 2[/itex], for any measurement result A and B.
When we take the mean value over all photon pairs, [itex]\,\left\langle S \right\rangle = \frac{1}{N}\sum\limits_{i = 1}^N {{S_j}} [/itex] we find it to be bounded, i.e.
[itex] - 2 \le \,\left\langle S \right\rangle \le 2[/itex].
This quantity is bounded whatever the values of A and B for the photon pairs.

Say that the choice of the angles [itex]a_i[/itex],[itex]b_i[/itex] is not random but they are correlated to each other. I don't see how this inequality could be violated by a local and realistic model.. Can you help? I am trying to understand how this *loophole* works..

The word "loophole" is not usually used in this context, as it has a rather different meaning altogether.

A local realistic model will always have S<=2, do you see that? (Since S is between -2 and +2.)

However, experiments typically give a value of S>2, often in the 2.2 to 2.4 range depending on the particulars of the setup and efficiency. The usual value given for the QM predicted theoretical value is about 2.7 (again this varies somewhat depending on assumptions). So quantum theory and experiment are in reasonable agreement, but are at odds with predictions based on local realistic assumptions.

Does that address your question?
 
  • #25
Thank you for your answer, although i don't think i understood your point.

You are saying that a local realistic model will always give |S|<=2.
So does this hold for any choice of the angles?
There is no demand for random non-correlated choice of angles?
Then what is all the fuss about free will of the observers etc and superdeterminism?

I thought that by using correlations between the choice of the angles, you could make a local realistic model violate the inequality.
Am i wrong?
 
  • #26
bohm2 said:
There seems to be only 3 options based on assumptions made by Bell:

1. Non-locality
2. Anti-realism
3. Superdeterminism (no freedom of choice)
Assuming either superdeterminism or nonlocality is not informative.

The answer is in the realm of anti-realism, which has to do with modeling restrictions.
 
  • #27
JK423 said:
You are saying that a local realistic model will always give |S|<=2.
So does this hold for any choice of the angles?
There is no demand for random non-correlated choice of angles?
Then what is all the fuss about free will of the observers etc and superdeterminism?

I thought that by using correlations between the choice of the angles, you could make a local realistic model violate the inequality.
Am i wrong?

At the risk of putting words in DrChinese's mouth (but if I get it wrong we'll find out:smile:), I expect that he meant "non-conspiratorial local realistic model". If the choice of angles is deterministic and the source knows what the determining rule is, or if the source is allowed to influence the choice of direction, then the source can produce pairs tailored to produce any results it pleases.

We tend to leave out the qualifier because the conspiratorial models are either uninteresting or reduce to some form of superdeterminism, or both.
 
  • #28
JK423 said:
Thank you for your answer, although i don't think i understood your point.

a) You are saying that a local realistic model will always give |S|<=2.
So does this hold for any choice of the angles?
There is no demand for random non-correlated choice of angles?

b) Then what is all the fuss about free will of the observers etc and superdeterminism?

c) I thought that by using correlations between the choice of the angles, you could make a local realistic model violate the inequality.
Am i wrong?

There are a batch of different issues in your comments. I will do my best to address.

a) Yes, LR models always yield |S|<=2 and that is for any choice of angles.

c) The LR model predicts results that are consistent with the inequality. However, those results do not happen in experimental situations. So the real world violates the inequality, not the LR model.

b) OK, the idea of superdeterminism is something I routinely criticize as non-scientific. But some folks I respect think it is worth mentioning, so I will attempt to describe the argument in as objective terms as able. I am answering this after c) so you can read c) again as needed.

The idea is that the angle settings that we think we are freely selecting correspond to ones in which the inequality will be violated, but that we are actually choosing ones in which this result was predetermined to violate the inequality even though the inequality is NOT really violated. So the results are predetermined, and further the results were predetermined in such a way to be misleading.

Imagine we are playing 3 card monte with queen and 2 jacks. You are trying to pick the queen each hand. I bend the queen card (but not the other 2) and shuffle them around. You pick the bent card and I turn it over to reveal a jack. You thought you freely chose the card but I fooled you ('cause I am sneaky). Every time we do it, you pick a jack. Eventually you conclude there is no queen. But actually there was one, you just didn't pick it.

Do you see that in this case, if there is something influencing you that you are not aware of, you might come to a false conclusion? This is the *analogy* to the superdeterminism argument. So all you have to do is acknowledge that IF your choice was somehow influenced with bias and you were not aware of it, THEN you could come to the wrong conclusion. This is superdeterminism.

----------------------

Keep in mind that I vehemently deny that random angle choices has anything to do with a Bell test OTHER THAN to enforce strict Einsteinian separability. That was demonstrated in 1998 by Weihs et al. In a normal Bell test, you do not need to do ANYTHING other than show that the cos^2(theta) relationship predicted by QM is consistent with your results. The multiple angles thing is not needed at all and tends to confuse everyone. The reason is that Bell demonstrated that LR theories will not be able to yield datasets consistent with QM's cos^2(theta). So if QM is right (confirmed experimentally) then LR cannot be.

You don't need to know anything about Bell's proof or inequality if you simply try to construct a local realistic dataset of your own (I supplied some example angles at the beginning of this thread). Just try to create a dataset and you will see it cannot ever match the cos^2(theta) at those angles. If QM is experimentally right about the cos^2 relationship, then LR is logically excluded.
 
  • #29
Nugatory said:
At the risk of putting words in DrChinese's mouth (but if I get it wrong we'll find out:smile:), I expect that he meant "non-conspiratorial local realistic model". If the choice of angles is deterministic and the source knows what the determining rule is, or if the source is allowed to influence the choice of direction, then the source can produce pairs tailored to produce any results it pleases.

We tend to leave out the qualifier because the conspiratorial models are either uninteresting or reduce to some form of superdeterminism, or both.

That's true, but I equate:

Conspiracy <=> Superdeterminism <=> Hand of god

in the sense that all of these are "outs". Of course these apply equally for all theories: evolution, cosmology, relativity, etc. I have no idea why any of these should be included as an qualification for a scientific discussion. You don't say "the universe is 13.7 billion years old UNLESS it is really 4000 years old and there is superdeterminism at work." I think everyone understands that if we are ALL being hoodwinked, then all bets are off on anything we might think we know.
 
  • #30
Nugatory said:
At the risk of putting words in DrChinese's mouth (but if I get it wrong we'll find out:smile:), I expect that he meant "non-conspiratorial local realistic model". If the choice of angles is deterministic and the source knows what the determining rule is, or if the source is allowed to influence the choice of direction, then the source can produce pairs tailored to produce any results it pleases.

We tend to leave out the qualifier because the conspiratorial models are either uninteresting or reduce to some form of superdeterminism, or both.

Hmm i think that this helped! Thank you.
So, if i understand correctly:

The reason why a local realistic (non-conspirational) model will always satisfy Bell's inequality is due to the factorization of [itex]S_j[/itex] (second line):
[itex]\begin{array}{l}
{S_j} = {A_j}\left( {{a_1}} \right){B_j}\left( {{b_1}} \right) + {A_j}\left( {{a_1}} \right){B_j}\left( {{b_2}} \right) + {A_j}\left( {{a_2}} \right){B_j}\left( {{b_1}} \right) - {A_j}\left( {{a_2}} \right){B_j}\left( {{b_2}} \right) \\
= {A_j}\left( {{a_1}} \right)\left( {{B_j}\left( {{b_1}} \right) + {B_j}\left( {{b_2}} \right)} \right) + {A_j}\left( {{a_2}} \right)\left( {{B_j}\left( {{b_1}} \right) - {B_j}\left( {{b_2}} \right)} \right) \\
= \pm 2 \\
\end{array}[/itex]. (1)
In order to violate Bell's inequality, this factorization should not be possible.
For example, non-locality would force the following change:
[itex]{A_j}\left( {{a_1}} \right) \cdot {B_j}\left( {{\beta _1}} \right) \to A_j\left( {{a_1},{\beta _1}} \right) \cdot {B_j}\left( {{a_1},{\beta _1}} \right)[/itex],
[itex]{A_j}\left( {{a_1}} \right) \cdot {B_j}\left( {{\beta _2}} \right) \to A_j\left( {{a_1},{\beta _2}} \right) \cdot {B_j}\left( {{a_1},{\beta _2}} \right)[/itex], (2)
etc.
This change allows the violation of the inequality because in general it's
[itex]A_j\left( {{a_1},{\beta _1}} \right) \ne A_j\left( {{a_1},{\beta _2}} \right)[/itex], (3)
preventing the factorization as in (1), so [itex]S_j=±2[/itex] won't be always true and a violation of Bell's inequality is possible.

Now in the case of superdeterminism and conspirational models (but still local and realistic), Eq. (3) seems to hold as well. If the source knows beforehand what is to be measured, it can prepare the photon A in such a way so that e.g.
[itex]A_j\left( {{a_1},{\beta _1}} \right) = + 1[/itex] and
[itex]A_j\left( {{a_1},{\beta _2}} \right) = - 1[/itex],
being different like in (3), having the potential to lead to a violation of Bell's inequality.

Hope that i got this right!Update: DrChinese i just saw your new post, thank you a lot for your detailed description, it clarifies lots of misconceptions that i had.
 
Last edited:
  • #31
DrChinese said:
That's true, but I equate:

Conspiracy <=> Superdeterminism <=> Hand of god

in the sense that all of these are "outs". Of course these apply equally for all theories: evolution, cosmology, relativity, etc.
I'm not sure if this is really the case. In classical theories, the state of a system remains unchange by measurements, because the interaction of the observer with the system can be made arbitrarily small. So the measurement outcome doesn't depend on the past of the observer because it doesn't depend on the observer at all. In QM, the interaction between the observer leads to a physical change of the state of the system. So at least the assumption that the measurement outcome doesn't depend on the past of the observer is not as easily justifiable as in a classical theory.
 
  • #32
kith said:
So at least the assumption that the measurement outcome doesn't depend on the past of the observer is not as easily justifiable as in a classical theory.

That argument strikes me as a bit of a red herring. Yes, a superdeterministic quantum theory must include the observer whereas a deterministic theory (whether classical or merely EPR-friendly) need not. But that doesn't make superdeterminism any more palatable1; and experiment has already rejected the deterministic observer-independent theories.

1: De [strike]gustibus[/strike] interpretationes non disputandum est.
 
  • #33
Nugatory said:
That argument strikes me as a bit of a red herring. Yes, a superdeterministic quantum theory must include the observer whereas a deterministic theory (whether classical or merely EPR-friendly) need not.
We know as a fact that measurements on a quantum mechanical system may change the state of this system, so every fundamental theory needs to include the observer somehow. This is a key difference between classical theories and QM. It isn't an argument for superdeterminism by itself. It simply shows that the question of superdeterminism has different implications in QM than in classical theories. In classical mechanics, the physics of the system is independent of the question wether the observer is free to chose what observables he wants to measure. In QM it's not.

I am not an advocate of superdeterminism. I just replied to a statement by DrChinese with which I don't agree.
 
  • #34
kith said:
I'm not sure if this is really the case. In classical theories, the state of a system remains unchange by measurements, because the interaction of the observer with the system can be made arbitrarily small. So the measurement outcome doesn't depend on the past of the observer because it doesn't depend on the observer at all. In QM, the interaction between the observer leads to a physical change of the state of the system. So at least the assumption that the measurement outcome doesn't depend on the past of the observer is not as easily justifiable as in a classical theory.

That does not make sense, kith. It doesn't matter whether a theory is classical or not! That is a completely arbitrary designation.

The fact is, there is no theory - now or ever - which explains how the observer's past has anything whatsoever to do with ANY experiment. That includes QM. It is just a blind ad hoc hypothesis thrown out by a few people. So you cannot explain WHY it should apply to entanglement more (or less) than the age of the universe or measurements of c or anything else.
 
  • #35
DrChinese said:
The fact is, there is no theory - now or ever - which explains how the observer's past has anything whatsoever to do with ANY experiment.
Usually, we have a system S and an observer O measuring some observable of the system. As soon as we consider the combined system S+O as a physical system (which may be observed by another observer O'), we acknowledge that the current state of S and O influences the evolution of the combined system. This evolution may of course include interactions between S and O. What's wrong with this kind of thinking?
 
<H2>1. Why are Bell's inequalities important in quantum mechanics?</H2><p>Bell's inequalities are important in quantum mechanics because they provide a way to test the validity of quantum theory against classical theories. The violation of these inequalities shows that quantum mechanics cannot be explained by classical theories and therefore highlights the unique nature of quantum systems.</p><H2>2. What is the significance of Bell's inequalities being violated?</H2><p>The violation of Bell's inequalities is significant because it demonstrates the existence of quantum entanglement, a phenomenon where two or more particles become connected in such a way that the state of one particle is dependent on the state of the other, regardless of the distance between them. This violates the principle of local realism and challenges our understanding of how the physical world operates.</p><H2>3. How do experiments show the violation of Bell's inequalities?</H2><p>Experiments designed to test Bell's inequalities involve measuring the correlation between the properties of two entangled particles. By comparing the results with the predictions of classical theories, researchers can determine if the inequalities are violated. Numerous experiments have been conducted, all of which have shown a violation of Bell's inequalities and therefore support the principles of quantum mechanics.</p><H2>4. Can Bell's inequalities be explained by hidden variables?</H2><p>No, Bell's inequalities cannot be explained by hidden variables. Hidden variables are theoretical properties that are not directly observable but are assumed to determine the outcome of experiments. However, the violation of Bell's inequalities has been confirmed through experiments, ruling out the possibility of hidden variables as an explanation.</p><H2>5. How do Bell's inequalities relate to the concept of non-locality?</H2><p>Bell's inequalities are closely related to the concept of non-locality, which refers to the ability of entangled particles to influence each other's properties instantaneously, regardless of the distance between them. The violation of Bell's inequalities is evidence of non-locality and challenges our understanding of causality in the physical world.</p>

1. Why are Bell's inequalities important in quantum mechanics?

Bell's inequalities are important in quantum mechanics because they provide a way to test the validity of quantum theory against classical theories. The violation of these inequalities shows that quantum mechanics cannot be explained by classical theories and therefore highlights the unique nature of quantum systems.

2. What is the significance of Bell's inequalities being violated?

The violation of Bell's inequalities is significant because it demonstrates the existence of quantum entanglement, a phenomenon where two or more particles become connected in such a way that the state of one particle is dependent on the state of the other, regardless of the distance between them. This violates the principle of local realism and challenges our understanding of how the physical world operates.

3. How do experiments show the violation of Bell's inequalities?

Experiments designed to test Bell's inequalities involve measuring the correlation between the properties of two entangled particles. By comparing the results with the predictions of classical theories, researchers can determine if the inequalities are violated. Numerous experiments have been conducted, all of which have shown a violation of Bell's inequalities and therefore support the principles of quantum mechanics.

4. Can Bell's inequalities be explained by hidden variables?

No, Bell's inequalities cannot be explained by hidden variables. Hidden variables are theoretical properties that are not directly observable but are assumed to determine the outcome of experiments. However, the violation of Bell's inequalities has been confirmed through experiments, ruling out the possibility of hidden variables as an explanation.

5. How do Bell's inequalities relate to the concept of non-locality?

Bell's inequalities are closely related to the concept of non-locality, which refers to the ability of entangled particles to influence each other's properties instantaneously, regardless of the distance between them. The violation of Bell's inequalities is evidence of non-locality and challenges our understanding of causality in the physical world.

Similar threads

Replies
4
Views
954
Replies
3
Views
774
Replies
71
Views
3K
Replies
72
Views
3K
Replies
0
Views
651
  • Quantum Physics
Replies
5
Views
1K
  • Quantum Physics
Replies
16
Views
2K
Replies
50
Views
4K
  • Quantum Physics
Replies
1
Views
700
Replies
25
Views
2K
Back
Top